Merge remote branch 'public/master'
[course.git] / latex / problems / Serway_and_Jewett_4 / problem22.16.tex
1 \begin{problem*}{22.16}
2 A wire $l = 2.80\U{m}$ in length carries a current of $I = 5.00\U{A}$
3 in a region where a uniform magnetic field has a magnitude of $B =
4 0.390\U{T}$.  Calculate the magnitude of the magnetic force on the
5 wire assuming that the angle between the magnetic field and the
6 current is
7  \Part{a} $\theta_a = 60.0\dg$,
8  \Part{b} $\theta_b = 90.0\dg$, and
9  \Part{c} $\theta_c = 120\dg$.
10 \end{problem*} % problem 22.16
11
12 \begin{solution}
13 Using our formula for the force on a wire due to a uniform field we
14 have
15 \begin{align}
16  \vect{F} &= I\vect{l}\times\vect{B} \\
17  F &= IlB\sin\theta \;,
18 \end{align}
19 so just pluggging in
20 \begin{align}
21  F_a &= IlB\sin\theta_a = \ans{4.73\U{N}}\\
22  F_b &= IlB\sin\theta_b = \ans{5.46\U{N}}\\
23  F_c &= IlB\sin\theta_c = \ans{4.73\U{N}} \;.
24 \end{align}
25 \end{solution}